Đến nội dung

Hình ảnh

[TOPIC] Hai bài toán mỗi ngày.


  • Please log in to reply
Chủ đề này có 173 trả lời

#21
tritanngo99

tritanngo99

    Đại úy

  • Điều hành viên THPT
  • 1644 Bài viết

Lời giải bài 15:

Đặt $w=x+y+z$

Khi đó theo giả thiết ta có: $\sum cos(x)=a.cos(w);\sum sin(x)=a.sin(w)$.

Khi đó: $\sum cos(y+z)=\sum cos(w-x)=\sum(cos(w).cos(x)+sin(w).sin(x))$.

$=(cos(w)).\sum(cos(x))+(sin(w)).\sum(sin(x))=a.cos^2(w)+a.sin^2(w)=a$.

Vậy ta có điều phải chứng minh.

Lời giải bài 16:

Ta có: $[x[x]]=1$.

$\implies 1\le x[x]<2$.

Đến đây ta xét các trường hợp sau:

a) $x\in(-\infty;-1)\implies [x]\le -1\implies x[x]>2\implies \text{ loại }$

b) $x=-1\implies [x]=-1\implies x[x]=1\implies [x[x]]=1\implies \text{ nhận }$.

c) $x\in(-1;0)\implies [x]=-1\implies x[x]=-x<1\implies \text{ loại }$.

d) $x\in [0;1)\implies [x]=0\implies [x[x]]=0\implies \text{ loại }$.

e) $x\in [1;2)\implies [x]=1\implies x[x]=x\implies [x[x]]=1\implies \text{ nhận }$.

f) $x\ge 2\implies [x]\ge 2\implies x[x]\ge 4\implies \text{ loại }$.

Vậy nghiệm của phương trình đã cho là: $x=-1$ hoặc $x\in \text{[1;2)}$



#22
tritanngo99

tritanngo99

    Đại úy

  • Điều hành viên THPT
  • 1644 Bài viết

Bài 17: Cho dãy số Fibonacci: $u_0=u_1=1$ và $u_{n+2}=u_{n+1}+u_n\forall n\ge 2$.

Chứng minh rằng: Trong $n^2-3n+4$ số đầu tiên của dãy luôn tồn tại một số chia hết cho $n$.

Bài 18: Cho các số nguyên dương $a,b,c,d$ thỏa mãn điều kiện: $a^2+ab+b^2=c^2+cd+d^2$. Chứng minh rằng: $a+b+c+d$ là hợp số.


Bài viết đã được chỉnh sửa nội dung bởi tritanngo99: 19-08-2018 - 04:45


#23
tritanngo99

tritanngo99

    Đại úy

  • Điều hành viên THPT
  • 1644 Bài viết

Lời giải bài 17: Xét các số $u_0$ đến $u_{n^2-3n+4}$ của dãy. Giả sử không có số nào trong các số này chia hết cho $n$.

Gọi $a_k$ là số dư của $u_k$ khi chia cho $n$ thì $a_k\in \text{{1;2;...;n-1}}$.

Xét các cặp $(a_k;a_{k+1})$ với $k=\overline{0,n^2-3n+4}$. Ta có: $n^2-3n+3$. Chú ý với $p>0$ mà $a_p=a_{p+1}$ ta suy ra: $a_{p-1}=a_{p+1}-a_p=0$. Dẫn đến $u_{p-1}$ chia hết cho $n$.

Do đó trong mỗi cặp thì các số khác nhau. Vì $a_{k}\in \text{{1;2;...;n-1}}$ và trong mỗi cặp $2$ số không giống nhau nên số giá trị các cặp có thể nhận được là $(n-1)(n-2)$.Theo nguyên lý Dirichlet suy ra phải có $2$ cặp nhận giá cặp giá trị giống nhau tức tồn tại $i,j$ mà $a_i=a_j,a_{i+1}=a_{j+1}$. Không mất tính tổng quát ta giả sử $i<j$ và $i$ là nhỏ nhất. Dễ thấy rằng nếu $i>0$ thì từ $(a_i;a_{i+1})=(a_j;a_{j+1})$ .Ta có thể suy ra được: $(a_{i-1};a_{i})=(a_{j-1};a_{j})$. Nhưng điều này trái với cách giả sử $i$ là số nhỏ nhất. Do đó $i=0$. Vì vậy ta được: $(a_{i};a_{i+1})=(a_{j};a_{j+1})=(1;1)\implies a_{j}=a_{j+1}=1$ với $j>0$. Điều này vô lý. Vậy điều giả sử ban đầu là sai. Ta suy ta điều phải chứng minh.

Lời giải bài 18:

Từ giả thiết suy ra: $(a+b)^2-ab=(c+d)^2-cd\iff (a+b+c+d)(a+b-c-d)=ab-cd$.

Giả sử ngược lại, $p=a+b+c+d$ là số nguyên tố. Thế thì từ $ab-cd=p(a+b-c-d)$, ta có:

$ab-cd\vdots p\implies ab-c(p-a-b-c)\vdots p\implies ab+c(a+b+c)\vdots p\implies (c+a)(c+b)\vdots p$.

Nhưng điều này là vô lý vì $p$ là số nguyên tố và $a,b,c,d>0$ nên $0<c+a,c+b<p$, suy ra $(c+a,p)=(c+b,p)=1,$ vậy không thể có $(c+a)(c+b)\vdots p$.

Vậy $a+b+c+d$ là hợp số.  

 


#24
tritanngo99

tritanngo99

    Đại úy

  • Điều hành viên THPT
  • 1644 Bài viết

Bài 19: Tìm tất các cặp số nguyên dương $(m,n)$ sao cho $\frac{m^4+n^2}{7^m-3^n}$ là một số nguyên.

Bài 20: Cho tứ giác lồi $ABCD$ sao cho $AB$ không song song với $CD$ và điểm $X$ nằm bên trong tứ giác thỏa: $\angle{ADX}=\angle{BCX}<90^0$ và $\angle{DAX}=\angle{CBX}<90^0$. Gọi $Y$ là giao điểm đường trung trực của $AB$ và $CD$. Chứng minh rằng: $\angle{AYB}=2\angle{ADX}$ 

 


#25
DOTOANNANG

DOTOANNANG

    Đại úy

  • ĐHV Toán Cao cấp
  • 1609 Bài viết

Lời giải bài $\left ( 18 \right )$. Ta có vai trò của $\left ( a,\,b \right ),\,\left ( c,\,d \right )$ đối xứng nên không mất tính tổng quát, ta giả sử: $a+ b< c+ d$ (bài toán hiển nhiên đúng khi $a+ b= c+ d$), nên: $a+ b+ c+ d= \frac{\left ( a+ c \right )\left ( b+ c \right )+ a^{2}+ b^{2}+ ab- c^{2}- cd- d^{2}}{a+ b- d}= \frac{\left ( a+ c \right )\left ( b+ c \right )}{a+ b- d}$ là một hợp số vì $\left ( a+ c \right )- \left ( a+ b- d \right )> 0,\,\left ( b+ c \right )- \left ( a+ b- d \right )> 0$

Kể cả khi tính đến $a+ b> c+ d$ thì không mất tính tổng quát, ta có thể giả sử: $c\geqq d$. Khi đó:

$$\left ( c- d \right )^{2}= 4\left ( c^{2}+ cd+ d^{2} \right )-3\left ( c+ d \right )^{2}> 4\left ( a^{2}+ ab+ b^{2} \right )- 3\left ( a+ b \right )^{2}= \left ( a- b \right )^{2}\Leftrightarrow  c>  d$$

nên: 

$a+ b+ c+ d= \frac{\left ( c- d+ a- b \right )\left ( c- d- a+ b \right )+ 4\left ( a^{2}+ ab+ b^{2}- c^{2}- cd- d^{2} \right )}{3\left ( a+ b- c- d \right )}$ $= \frac{\left ( c- d+ a- b \right )\left ( c- d- a+ b \right )}{3\left ( a+ b- c- d \right )}$

là một hợp số vì:

$\left ( c+ d- a- b \right )- 3\left ( a+ b- c- d \right )$ $= \frac{4\left ( c^{2}+ cd+ d^{2}- a^{2}- ab- b^{2} \right )+ \left ( c- d+ a- b \right )^{2}+ 3\left ( a+ b- c- d \right )\left ( a+ b- c+ 3\,d \right )}{2\left ( c- d \right )}> 0$

$\left ( c- d- a+ b \right )- 3\left ( a+ b- c- d \right )$ $= \frac{4\left ( c^{2}+ cd+ d^{2}- a^{2}- ab- b^{2} \right )+ \left ( c- d- a+ b \right )^{2}+ 3\left ( a+ b- c- d \right )\left ( a+ b- c+ 3\,d \right )}{2\left ( c- d \right )}> 0$

 

 



#26
DOTOANNANG

DOTOANNANG

    Đại úy

  • ĐHV Toán Cao cấp
  • 1609 Bài viết

Lời giải bài $\left ( 14 \right )$. Giả sử với $a^{2}+ b^{2}<  1$ thì phương trình $\left ( a^{2}+ b^{2}- 1 \right )x^{2}- 2\left ( ac+ bd- 1 \right )x+ \left ( c^{2}+ d^{2}- 1 \right )= 0$ có nghiệm. Thật vậy, khi đó:

$\Delta _{x}= \frac{\left ( dab- b^{2}c- a+ c \right )^{2}+ \left ( b- d \right )^{2}\left ( 1- a^{2}- b^{2} \right )}{1- b^{2}}\geqq 0$ (điều này vô lí với giả thiết), tương tự ta có: $c^{2}+ d^{2}>  1$.

[điều phải chứng minh!] [tam thức bậc hai]



#27
tritanngo99

tritanngo99

    Đại úy

  • Điều hành viên THPT
  • 1644 Bài viết

Lời giải bài 19:

Đặt $A=\frac{m^4+n^2}{7^m-3^n}$.

Dễ thấy $7^m$ và $3^n$ lẻ với mọi $m,n$ nguyên dương nên suy ra $7^m-3^n$ chẵn nên để $A$ nguyên thì $m,n$ phải cùng tính chẵn lẻ.

Ta xét 2 trường hợp:

TH1: $m,n$ lẻ, ta có: $7^m\equiv 3^{n}\equiv (-1)(\text{ mod 4 })$ và $m^2-1=(m-1)(m+4)\vdots 4$ nên $m^2\equiv 1(\text{ mod 4})$. Chứng minh tương tự,ta có: $n^2\equiv 1(\text{ mod 4})$. Suy ra: $m^4+n^2\equiv 2(\text{ mod 4})$.

(Loại vì tử chia hết cho $4$, mẫu không chia hết cho $4$).

TH2: $m,n$ cùng chẵn, ta có $7^m\equiv 3^n\equiv 1(\text{ mod 8})$. Nên để $A$ nguyên thì $m^4+n^2\vdots 8$ mà $m^4\vdots 16$ nên $n^2\vdots 8\implies n\equiv 4$. Vậy $m=2a,n=4b(a,b\in \mathbb{N^*})$.

Khi đó: $A=\frac{16(a^4+b^2)}{7^{2a}-9^{2b}}$.

Do $A$ nguyên và $A\ne 0$ nên $|A|\ge 1$. Suy ra: $\left| \frac{16(a^4+b^2)}{(7^{a}+9^{b})(7^a-9^b)}\right|\ge 1$.

$\implies \frac{16(a^4+b^2)}{7^a+9^b}\ge |7^a-9^b|$.

Mặt khác: $7^{a}-9^{b}$ là số nguyên chẵn và $7^a\ne 9^b$ nên $|7^{a}-9^{b}|\ge 2$.

Suy ra: $16(a^4+b^4)\ge 2(7^a+9^b)>2(7^a+7^b)$.

Đặt $c=\text{ max(a,b)}$, ta có:

$16c^4+16c^2\ge 16(a^4+b^2)>2(7^a+7^b)>2.7^c(*)$.

Ta chứng minh: $2.7^c>16c^4+16c^2\forall c\ge 4,c\in \mathbb{N}$ bằng quy nạp.

+ Dễ dàng kiểm chứng mệnh đề đúng khi $c=4$.

+ Giả sử mệnh đề đúng khi $c=N\ge 4$, ta sẽ chứng minh mệnh đề đúng khi $c=N+1$. Thật vậy:

$2.7^{N}>16N^4+16N^2$.

$\implies 2.7^{N+1}>7(16N^4+16N^2)$.

$\implies 2.7^{N+1}-16(N+1)^4-16(N+1)^2>16(7N^4+7N^2-(N+1)^4-(N+1)^2)$.

$\implies 2.7^{N+1}-16(N+1)^4-16(N+1)^2>16[N^3(N-4)+N(N^3-6)+N^4-2+3N^4]>0$

(Đúng do $N\ge 5$).

Vậy $2.7^c>16c^4+16c^2\forall c\ge 4,c\in \mathbb{N}$.

Từ $(*)$ ta suy ra: $1\le c\le 3\implies 1\le a,b\le 3$. Thử các bộ giá trị $(a,b)$ với $1\le a,b\le 3$, ta nhận: $(a,b)=(1,1)$.

Thử lại, ta nhận: $(m,n)=(2,4)$.

***********************************************************************************************

Lời giải bài 20:vbv.jpg

Bổ đề: Cho hai đường tròn $(O_1)$ và $(O_2)$ cắt nhau tại $X,Z$. Lấy $A$ là một điểm bất kỳ nằm trên $(O_1)$. Dựng tia $ZB$ đối xứng tia $ZA$ qua $ZX$ với $B$ thuộc $(O_2)$. Gọi $O$ là tâm đường tròn ngoại tiếp $\triangle{ABZ}$. Khi đó ta có: $OO_1=OO_2$.

Chứng minh bổ đề:

Ta có: $(OO_1,O_1O_2)\equiv (OO_1,AZ)+(AZ,ZX)+(ZX,O_1O_2)$.

$\equiv (O_1O_2,ZX)+(ZX,ZB)+(ZB,OO_2)$

$\equiv (O_1O_2,OO_2)(\text{ mod }\pi)$.

Do đó tam giác $OO_1O_2$ cân tại $O$ nên $OO_1=OO_2$.

Bổ đề được chứng minh.

Trở lại bài toán:

Gọi $(O_1),(O_2)$ lần lượt là đường tròn ngoại tiếp $\triangle{XAD},\triangle{XBC}$.

Gọi $Z$ là giao điểm thứ $2$ của $(O_1),(O_2)$.

Gọi $(O),(O')$ lần lượt là đường tròn ngoại tiếp $\triangle{ZAB}$ và $\triangle{ZCD}$.

Gọi $Y'$ là giao điểm thứ $2$ của $(O),(O')$.

Ta có: $M=ZX\cap (O)(M\ne Z); N=ZX\cap (O')(N \ne Z)$.

Ta có: $(ZA,ZX)\equiv (DA,DX)\equiv (CX,CB)\equiv (ZX,ZB)(\text{ mod }\pi)$. Nên áp dụng bổ để trên ta có: $OO_1=OO_2$.

Tương tự: $O'O_1=O'O_2$.

Suy ra: $OO'\bot O_1O_2$.

Mặt khác: $XZ\bot O_1O_2,ZY'\bot OO'$ nên $ZY'\bot ZX$.

Xét $(O)$ có $ZY'\bot ZM$ và $M$ là điểm chính giữa cung $AB$ không chứa $Y'$, ta suy ra $Y'A=Y'B$.

Tương tự: $Y'C=Y'D$ nên $Y'\equiv Y$.

Vì vậy: $\angle{AYB}=\angle{AZB}=2\angle{ADX}$. 

Ta có điều phải chứng minh.


Bài viết đã được chỉnh sửa nội dung bởi tritanngo99: 20-08-2018 - 05:40


#28
tritanngo99

tritanngo99

    Đại úy

  • Điều hành viên THPT
  • 1644 Bài viết

Bài 21: Tìm tất cả các đa thức có hệ số thực thỏa mãn: $P(x).P(x+1)=P(x^2+2)\forall x\in \mathbb{R}$.

Bài 22: Chứng minh rằng với mỗi số nguyên dương $r$ nhỏ hơn $59$ đều tồn tại duy nhất số nguyên dương $n$ nhỏ hơn $59$ sao cho $2^{n}-r$ chia hết cho $59$

 


#29
Hr MiSu

Hr MiSu

    Thượng sĩ

  • Thành viên
  • 206 Bài viết

Bài 21: Tìm tất cả các đa thức có hệ số thực thỏa mãn: $P(x).P(x+1)=P(x^2+2)\forall x\in \mathbb{R}$.

Có vẻ topic hơi buồn, mình xin đóng góp ý kiến cho bài toán này:

Thực chất đây là 1 trường hợp nhỏ trong dạng toán tổng quát, có thể phát biểu thành định lý như sau:

Xét pth đa thức: $P(f(x))P(g(x))=P(h(x))$,

Nhận xét 1: với các đa thức $f,g,h$ có hệ số thực thỏa mãn $deg(f)+deg(g)=deg(h)$ và thỏa mãn 1 trong 2 điều kiện sau:

1) $deg(f) \neq deg(g)$

2) $deg(f)=deg(g)$ và $f*+g* \neq 0$ trong đó $f*,g*$ tương ứng là hệ số bậc cao nhất của $f,g$

Thì khi đó với mọi số nguyên dương $n$ tồn tại nhiều nhất 1 đa thức thỏa mãn.

Nhận xét 2: Nếu $P(x)$ thỏa mãn thì $(P(x)^n$ cũng thỏa mãn.

Chứng minh nhận xét 1: 

Giả sử P là đa thức bậc n t/m đề, P*,f*,g*,h* t/ư là hệ số bậc cao nhất của P,f,g,h.

Ta có: $P*=(\frac{h*}{f*.g*})^n$ suy ra nếu tồn tại Q(x) bậc n cũng t/m đề thì $Q*=P*$

Đặt $Q(x)=P(x)+R(x), 0 \leq deg(R) <n$

-> $P(f)R(g)+R(f)P(g)+R(f)R(g)=R(h)$

TH1: $deg(f) \neq deg(g)$. Gs $deg(f)>deg(g)$ -> vế trái có bậc $ndeg(f)+rdeg(g)> rdeg(h)$ là bậc vế phải vô lý!!

TH2: Dễ thấy bậc vế trái cũng vẫn là $ndeg(f)+rdeg(g)$ suy ra vô lý.

Chứng minh nhận xét 2: Hiển nhiên dựa vào tính nhân đa thức.


s2_PADY_s2

Hope is a good thing, maybe the best thing, and no good thing ever dies


#30
Hr MiSu

Hr MiSu

    Thượng sĩ

  • Thành viên
  • 206 Bài viết

Bài 22: Chứng minh rằng với mỗi số nguyên dương $r$ nhỏ hơn $59$ đều tồn tại duy nhất số nguyên dương $n$ nhỏ hơn $59$ sao cho $2^{n}-r$ chia hết cho $59$

Để ý 59 là snt,

Bổ đề 1: $ord_{59}(2)=58$ 

Thật vậy theo fecmat nhỏ ta có: $ord_{59}(2)|58=2.29$ ,thử sẽ ra

Bổ đề 2: $\left \{ 2^i|i=1,2,...,59 \right \}$ lập thành HTDĐĐ mod 59

Thật vậy, nếu tồn tại $i > j$ sao cho $2^i\equiv 2^j (mod 59) \Rightarrow 2^{i-j}\equiv 1 (mod 59)$

Theo bổ đề 1: $i-j\geq 58\Rightarrow i=59,j=1$ vô lý vậy ta có đpcm\

Trở lại bài toán:Hiển nhiên rồi :D


Bài viết đã được chỉnh sửa nội dung bởi Hr MiSu: 20-08-2018 - 12:21

s2_PADY_s2

Hope is a good thing, maybe the best thing, and no good thing ever dies


#31
tritanngo99

tritanngo99

    Đại úy

  • Điều hành viên THPT
  • 1644 Bài viết

Lời giải bài 21: 

Với đa thức hằng $P(x)\equiv a$, ta có: $a^2=a$ nên $a=0$ hoặc $a=1$.

Ta thấy $P(x)\equiv 0$ và $P(x)\equiv 1$ thỏa mãn bài ra.

Xét trường hợp $P(x)$ khác hằng số. Ta có các nhận xét sau:

+ Đa thức bậc hai $P(x)=x^2-x+2$ thỏa mãn bài ra.

+Nếu $P(x)$ thỏa mãn bài toán thì hiển nhiên mọi đa thức dạng: $Q(x)=(P(x))^n(n\in \mathbb{N^*})$ cũng thỏa mãn bài ra.

+Đa thức $P(x)$ thỏa mãn bài toán thì hệ số bậc cao nhất bằng $1$.

+ Đa thức $P(x)$ thỏa mãn bài toán thì nó không thể có nghiệm thực. Thật vây, nếu $x_0$ là nghiệm thì dãy số tăng $x_1=x_0^2+2,x_2=x_1^2+2,...$ cũng là nghiệm, mâu thuẩn.

Vậy bậc của $P(x)$ là một số chẵn, $degP(x)=2m(m\in \mathbb{N^*})$, đặt

$P(x)=(x^2-x+2)^m+Q(x),degQ(x)=q<2m$.

Thế vào điều kiện bài ra, ta thu được:

$(x^2-x+2)^mQ(x)+(x^2-x+2)^mQ(x+1)+Q(x)Q(x+1)=Q(x^2+2)(1)$.

Ta chứng minh $Q(x)\equiv 0$.

Thật vậy, giá sử $Q(x)$ không đồng nhất bằng $0$.

Ta thấy rằng: Bậc của đa thức ở vế trái $(1)$ bằng $2m+q$, còn bậc ở vế phải bằng $2q$.

So sánh bậc của hai vế, ta thu được: $q=2m$. Điều này là vô lý.

Vậy $Q(x)\equiv 0$ và $P(x)=(x^2-x+2)^m$.

Kết luận: Các đa thức thỏa mãn điều kiện bài toán là $P(x)\equiv 0,P(x)\equiv 1,P(x)=(x^2-x+2)^m,m\in \mathbb{N^*}$.

*****************************************************************************************

Lời giải bài 22:

Vì $59$ là số nguyên tố nên theo định lý Fermat nhỏ ta có: $2^{58}-1\equiv 0(\text{ mod 59})$.

Giả sử $k$ là số nguyên dương nhỏ nhất thỏa mãn: $2^{k}-1\equiv 0(\text{ mod 59})$, thế thì $k\le 58$.

Ta chứng minh $k$ là ước của $58$.Thật vậy, giả sử r là số dư khi chia $58$ cho $k$, nghĩa là $58=ak+r$ với $a,r$ là số tự nhiên và $r<k$.

Ta có: $2^{k} \equiv 1(\text{ mod 59})$ nên $2^{58}=2^{ak+r}\equiv 2^{r}\equiv 1(\text{ mod 59})$.

Suy ra: $2^{r}-1\equiv 0(\text{ mod 59})$.

Vì giả thiết $k$ là số nguyên dương nhỏ nhất có tính chất trên nên $r=0$, do đó $k$ là ước của $58$.

Vậy $k$ chỉ có thể nhận giá trị trong tập hợp $\text{{1,2,29,58}}$.

Nếu $k=1$ hoặc $k=2$ thì $2^{k}-1$ không chia hết cho $59$.

Xét $k=29$,ta có: $2^{7}=128\equiv 10(\text{ mod 59})$.

$\implies 2^{28}\equiv 10^{4}\equiv 29(\text{ mod 59})$.

$\implies 2^{29}\equiv 58(\text{ mod 59})\implies 2^{29}-1\equiv 57(\text{ mod 59})$.

Do đó: $2^{29}-1$ không chia hết cho $59$.

Vậy số nguyên dương nhỏ nhất $k$ sao cho $2^{k}-1$ chia hết cho $59$ là $58$.

Bây giờ giả sử có hai số dương $a,b$ sao cho $a<b<59$ và $2^{a}-2^{b}$ có cùng số dư khi chia cho $59$.

Thì $2^b-2^a=2^a(2^{b-a}-1)\vdots 59$.

Điều này không thể xảy ra vì $b-a$ là số nguyên dương nhỏ hơn $58$ và $(2,59)=1$.

Ta được: $2^1,2^2,...,2^{58}$ khi chia cho $59$ được $58$ số dư khác nhau và khác $0$.

Vậy với mỗi số nguyên dương $r$ nhỏ hơn $59$ đều tồn tại số nguyên dương $n$ nhỏ hơn $59$ sao cho $2^{n}$ và $r$ có cùng 

số dư khi chia cho $59$, hay $(2^{n}-r)$ chia hết cho $59$.


Bài viết đã được chỉnh sửa nội dung bởi tritanngo99: 21-08-2018 - 04:47


#32
tritanngo99

tritanngo99

    Đại úy

  • Điều hành viên THPT
  • 1644 Bài viết

Bài 23: Chứng minh rằng tổng: $S_n=\sum\limits_{k=0}^{n}\frac{1}{C_{n}^k}$ có giới hạn khi $n$ tăng lên vô hạn và tìm giới hạn đó.

Bài 24: Tìm số tự nhiên lẻ $n$ nhỏ nhất sao cho $n^2$ biểu diễn được thành tổng của một số lẻ các số chính phương liên tiếp.

 


#33
Hr MiSu

Hr MiSu

    Thượng sĩ

  • Thành viên
  • 206 Bài viết

Bạn dậy sớm ghê, lúc đó mình ngủ đc 1 tiếng rồi, ngại đánh latex nên đánh tạm vào word :)

Capture.png


Bài viết đã được chỉnh sửa nội dung bởi Hr MiSu: 21-08-2018 - 12:13

s2_PADY_s2

Hope is a good thing, maybe the best thing, and no good thing ever dies


#34
canletgo

canletgo

    Sĩ quan

  • Thành viên
  • 389 Bài viết

Bài 23: 

Hình gửi kèm

  • DSC_0025.JPG

Bài viết đã được chỉnh sửa nội dung bởi canletgo: 21-08-2018 - 13:46

Alpha $\alpha$ 


#35
tritanngo99

tritanngo99

    Đại úy

  • Điều hành viên THPT
  • 1644 Bài viết

Lời giải bài 23:

Áp dụng hệ thức quen thuộc: $C_{n}^{k}=C_{n}^{n-k}(n\in \mathbb{N^*};k\le n)$.

Ta có: $S_{2n}=\sum\limits_{k=0}^{2n}\frac{1}{C_{2n}^{k}}=2\sum\limits_{k=0}^{n-1}\frac{1}{C_{2n}^{k}}+\frac{1}{C_{2n}^n}$

$=2+\frac{2}{C_{2n}^{1}}+2\sum\limits_{k=2}^{n-1}+\frac{1}{C_{2n}^n}$.

Mặt khác, ứng với $2\le k\le n$, thì:

$C_{2n}^{k}=\frac{(2n)!}{k!(2n-k)!}\ge n(2n-1)$.

Nên $2<S_{2n}\le 2+\frac{2}{2n}+\frac{2(n-2)}{n(2n-1)}+\frac{1}{n(2n-1)}$.

hay $2<S_{2n}\le 2+\frac{2}{n}+\frac{2n-3}{n(2n-1)}$.

Ta lại có: $lim_{n\rightarrow \infty}(\frac{2}{2n}+\frac{2n-3}{n(2n-1)})=0$.

nên $lim_{n\rightarrow \infty}S_{2n}=2$.

Tương tự, từ hệ thức: 

$S_{2n+1}=\sum\limits_{k=0}^{2n+1}\frac{1}{C_{2n}^{k}}=2\sum\limits_{k=0}^{n}\frac{1}{C_{2n+1}^{k}}$.

$2+\frac{2}{C_{2n+1}^{2}}+2\sum\limits_{k=2}^n\frac{1}{C_{2n+1}^{k}}$.

Mặt khác, ứng với $2\le k\le n$ thì

$C_{2n+1}^{k}=\frac{(2n+1)!}{k!(2n+1-k)!}\ge n(2n+1)$.

nên $2<S_{2n+1}\le 2+\frac{2}{2n+1}+\frac{2(n-1)}{n(2n+1)}$.

Ta lại có: $lim_{n\rightarrow \infty}(\frac{2}{2n+1}+\frac{2(n-1)}{n(2n+1)})=0$.

nên $lim_{n\rightarrow \infty}S_{2n+1}=2$.

Từ đây ta suy ra điều phải chứng minh.

*****************************************************************************************************

Lời giải bài 24:

Giả sử số lẻ $n^2$ biểu diễn được thành $2k+1$ số chính phương liên tiếp với $k>0$, nghĩa là: 

$n^2=(a-k)^2+(a-k+1)^2+...+(a+k)^2$.

Hay $n^2=(2k+1)a^2+2(1^2+2^2+...+k^2)(1)$.

Áp dụng hệ thức quen thuộc: $1^2+2^2+...+k^2=\frac{k(k+1)(2k+1)}{6}$

Nên từ $(1)$ suy ra: $3n^2=3(2k+1)a^2+k(k+1)(2k+1)(2)$.

Chú ý rằng: Với $n$ lẻ thì:

$n^2=(2t+1)^2=4t(t+1)+1=8s+1$ với $t,s$ nguyên.

Vì $a^2$ cũng lẻ nên $a^2=8r+1$ với $r$ nguyên.

Từ đó và từ $(2)$ suy ra: $3(8s+1)=3(2k+1)(8s+1)+k(k+1)(2k+1)$.

Suy ra: $6k+k(k+1)(2k+1)=k[6+(k+1)(2k+1)]$ chia hết cho $8$.(3)

a) Nếu $k$ chẵn thì $6+(k+1)(2k+1)$ lẻ nên $k$ phải chia hết cho $8$.

b) Nếu $k$ lẻ, đặt $k=8p+r$ với $r=1,3,5,7$.

Thay vào  $(3)$ ta được: $6+(8k+r)(16k+2r+1)=8v+6+(r+1)(2r+1)$ chia hết cho $8$ (với $v$ nguyên). 

Thử với $r=1,3,5,7$ chỉ có: $r=5$ thỏa mãn. Vậy $k=8p+5$.

+Xét $k=5$. Từ $(2)$ có $n^2=11a^2+110$ hay $n^2=11(a^2+10)(4)$.

Suy ra: $a^2+10=a^2-1+11=(a-1)(a+1)+11$ chia hết cho $11$.

Chú ý rằng, số lẻ $a>k=5$ nên thử với  $a=21,23,...$ thấy $a=23,n=77$.thỏa mãn $(4)$.

Còn với $k=8p+5\ge 13$ thì:

$n^2\ge (2k+1)k^2+\frac{k(k+1)(2k+1)}{3}>77^2$.

Xét $k=8$. Từ $(1)$ có $n^2=17a^2+17.24(5)$.

Phương trình $(5)$ không có nghiệm nguyên.

Thật vậy:

+nếu $n\vdots 3$ thì $a$ phải chia hết cho $3$,

khi đó $n^2\vdots 9$ nhưng vế phải của $(4)$ không chia hết cho $9$.

+nếu $n=3m+r,(r=1,2)$ thì $n^2\equiv 1(mod 3)$ nhưng vế phải của $(5)$ chia $3$ dư 2.

Còn với $k=8p\ge 16\implies n^2>77^2$.

  Vậy số tự nhiên $n$ nhỏ nhất thỏa mãn đề bài là $77$, lúc đó ta có:

$77^2=18^2+19^2+...+27^2+28^2$. Tổng này có $11$ số chính phương liên tiếp.


Bài viết đã được chỉnh sửa nội dung bởi tritanngo99: 22-08-2018 - 05:20


#36
tritanngo99

tritanngo99

    Đại úy

  • Điều hành viên THPT
  • 1644 Bài viết

Bài 25: Trên mặt phẳng, cho hai điểm cố định $M,N$ và tam giác $ABC$ có $BC<MN$ và $\angle{BAC}<90^0$. Cho tam giác $ABC$ chuyển động trượt trên mặt phẳng sao cho độ dài ba cạnh $AB,BC,CA$ không đổi, đường thẳng $AB$ đi qua $M$ và đường thẳng $AC$ đi qua $N$. Chứng minh rằng đường thẳng $BC$ luôn luôn tiếp xúc với một đường tròn cố định.

Bài 26: Tìm tất cả các cặp số nguyên dương $a,b$ sao cho $\frac{a^2-2}{ab+2}$ là số nguyên.

 


#37
tritanngo99

tritanngo99

    Đại úy

  • Điều hành viên THPT
  • 1644 Bài viết

Lời giải bài 25:

 Tù giả thiết ta có $4$ trường hợp xảy ra:

$M\in Ax,N\in Ay(1)$;

$M\in Ax';N\in Ay'(2)$;

$M\in Ax';N\in Ay(3)$;

$M\in Ax,N\in Ay'(4)$;

Trong $2$ trường hợp $(1)$ và $(2)$ do $\angle{BAC}=\alpha$ không đổi và $M,N$ cố định nên đỉnh $A$ chạy trên cung chứa góc $\alpha$ vẽ trên đoạn $MN$.

Trong $2$ trường hợp $(3)$ và $(4)$ đỉnh $A$ chạy trên cung chứa góc $180^0-\alpha$ vẽ trên đoạn $MN$ và khác phía với cung nói trên.

                                                                    cvb.jpg

Vậy $A$ chạy trên đường tròn $(O)$ cố định.

Ta chỉ cần xét $1$ trường hợp đầu, các trường hợp còn lại tương tự. Từ $A$ kẻ đường thằng song song với $BC$ cắt đường tròn $(O)$ tại $I$.Kẻ đường cao $AH$ của $\triangle{ABC}$. Từ $I$ kẻ $IK\bot BC$. Vì $\angle{IAN}$ bằng hoặc bù với $\angle{ACB}$ không đổi và $N$ cố định nên $I$ cố định và $IK=AH=h$ không đổi. Vậy $BC$ tiếp xúc với đường tròn cố định tâm $I$ bán kính $h$.

 

Lời giải bài 26:

Dễ thấy không có cặp số nguyên dương $(a,b)$ nào mà $a=1$ thỏa mãn đề bài. Do đó phải có $a\ge 2$.

Từ giả thiết suy ra:$(a^2-2)\vdots (ab+2)\implies b(a^2-2)\vdots (ab+2)$.

$\implies a(ab+2)-2(a+b)\vdots (ab+2)$.

$\implies 2(a+b)\vdots (ab+2)$.

$\implies 2(a+b)=k(ab+2)(k\in \mathbb{N^*})$.

+ Nếu $k=1$, ta có:

$2(a+b)=ab+2\iff (a-2)(b-2)=2$.

Do $a-2\ge 0;b-2\ge 0$ nên chỉ xảy ra:

$(a;b)=(3;4);(4;3)$.

Thử lại chỉ có: $(a;b)=(4;3)$ là thỏa mãn đề bài.

Nếu $k\ge 2$ ta có: $2(a+b)=k(ab+2)\ge 2(ab+2)$.

$\implies a+b\ge ab+2\implies (a-1)(b-1)+1\le 0$. Điều này không xảy ra.

Vậy chỉ có cặp số: $(a,b)=(4,3)$ thỏa mãn đề bài.  


Bài viết đã được chỉnh sửa nội dung bởi tritanngo99: 23-08-2018 - 05:57


#38
tritanngo99

tritanngo99

    Đại úy

  • Điều hành viên THPT
  • 1644 Bài viết

Bài 27: Tìm tất cả các số nguyên dương $x,y,z$ thỏa mãn $x^2+y^2=z^2$ biết rằng $z$ là một số tự nhiên lẻ và $x,y$ là lũy thừa của một số nguyên tố.

Bài 28: Điền $29$ số nguyên dương đầu tiên vào các ô vuông con của bảng $6x5$ như sau:

 1.jpg

Cho phép thay đổi vị trị của các số trong bảng theo quy tắc: Mỗi lần, lấy một số nằm ở ô kề với ô trống rồi chuyển số đó sang ô trống.

  Hỏi nhờ việc thực hiện liên tiếp một số lần hữu hạn phép chuyển số nói trên đối với bảng số ban đầu ta có thể nhận được bảng số sau hay không: 2.jpg

 


#39
tritanngo99

tritanngo99

    Đại úy

  • Điều hành viên THPT
  • 1644 Bài viết

Lời giải bài 27: 

 Vì $z$ lẻ suy ra $x$ và $y$ khác tính chẵn lẻ. Không giảm tổng quát, giả sử $x$ chẵn, $y$ lẻ. Theo giả thiết: $x=2^{k},y=p^{t}$(p lẻ, nguyên tố).

Ta có: $x^2=2^{2k}=(z-y)(z+y)$.

Vậy $z-u=2^{u};z+u=2^{v};u+v=2k,u<v$.

Suy ra: $y=2^{v-1}-2^{u-1}$.

Vì $y$ lẻ nên $u=1$ và $y=2^{2k-2}-1=(2^{k-1}-1)(2^{k-1}+1)=p^{t}$.

Nếu $2^{k-1}-1>1$ thì $2^{k-1}-1\vdots p$ và $2^{k+1}+1\vdots p\implies 2\vdots p\implies $ mâu thuẩn.

Thành thử: $2^{k-1}-1=1\implies k=2\implies x=4,y=3,z=5$.

Vậy ta có nghiệm: $(3,4,5),(4,3,5)$.

Lời giải bài 28:

Giả sử, nhờ phép chuyển số theo quy tắc của đề bài, từ bảng 1 ta có thể nhận được bảng 2(*). Ta coi ô trống của mỗi bảng là ô được điền số 0. Với mỗi bảng số nhận được trong quá trình chuyển số, ta liệt kê tất cả các số trong bảng theo thứ tự từ trái qua phải, từ trên xuống dưới. Khi đó, ứng với mỗi bảng số ta sẽ có một hoán vị của $30$ số tự nhiên đầu tiên. Và do đó, giả sử $(*)$ cho thấy: Từ hoán vị $(1,2,3,4,5,6,7,8,9,10,11,12,0,13,14,15,16,17,18,19,20,21,22,23,24,25,26,27,28,29)$( gọi là hoán vị I) ta có thể nhận đươc hoán vị $(29,2,3,4,5,6,7,8,9,10,11,12,0,13,14,15,16,17,18,19,20,21,22,23,24,25,26,27,28,1)$ (gọi là hoán vị II) nhờ việc thực hiện liên tiếp một số hữu hạn lần phép đổi chỗ các số hạng trong hoán vị theo quy tắc: mỗi lần, lấy một số hạng khác $0$ của hoán vị rồi đổi vị trí của số hạng đó và $0$ cho nhau(1).

Giả sử $(a_1,a_2,...,a_{30})$ là một hoán vị của $30$ số tự nhiên đầu tiên. Ta gọi cặp số $a_i,a_j$ là cặp số ngược của hoán vị vừa nêu nếu $a_i>a_j$ và $i<j$. Dễ thấy, sau một lần thực hiện phép đổi chỗ các số hạng theo quy tắc $(1)$ đối với hoán vị $(a_1,a_2,...,a_{30})$ thì số cặp số ngược của hoán vị đó sẽ tăng hoặc giảm một số lẻ đơn vị.(2).

Ta có: Số cặp số ngược của hoán vị $(I)$ là $12$ và số cặp số ngược của hoán vị $(II)$ là $67$. Từ đó kết hợp với $(2)$ suy ra từ hoán vị $I$ ta chỉ có thể nhận được hoán vị $II$ sau một số lẻ lần thực hiện phép đổi chỗ các số hạng. Điều này cho thấy, nếu từ bảng 1 ta nhận được bảng 2 thì số lần chuyển số phải là số lẻ (3).

Tô màu tất cả các ô vuông con của bảng $6x5$ bởi hai màu xanh, đỏ sao cho hai ô kề nhau có màu khác nhau. Thế thì, sau mỗi lần chuyển số , số $0$ sẽ được chuyển từ ô có màu này sang ô có màu kia. Và vì thế, do số $0$ ở bảng 1 và số $0$ ở bảng 2 nằm ở hai ô cùng màu nên từ bảng 1 ta chỉ có thể nhận được bảng 2 sau một số lần chẵn chuyển số. Điều này mẫu thuẩn với $(3)$ và mâu thuẩn đó cho thấy: Từ bảng 1 ta không thể nhận được bảng 2 nhờ phép chuyển số theo quy tắc của đề bài. 


Bài viết đã được chỉnh sửa nội dung bởi tritanngo99: 24-08-2018 - 05:36


#40
tritanngo99

tritanngo99

    Đại úy

  • Điều hành viên THPT
  • 1644 Bài viết

Bài 29: Xét dãy số $(u_n)(n=1,2,...)$ được xác định bởi: $u_1=a>0,u_{n+1}=u_n+[\sqrt{u_n}]-[\frac{u_n}{3}]$ với mọi $n=1,2,3,...$.

Chứng minh rằng: Dãy số này có giới hạn và tìm giới hạn đó.

Bài 30: Tìm tất các hàm số $f:\mathbb{Q}\to \mathbb{Q}$ thỏa mãn: $f(f(x)+y)=x+f(y)$ với mọi số $x,y$ thuộc tập hợp số hữu tỉ $\mathbb{Q}$.

 





0 người đang xem chủ đề

0 thành viên, 0 khách, 0 thành viên ẩn danh